sábado, 13 de junio de 2015



1- (IMO 1996) En el hexágono convexo ABCDEF las parejas de lados AD con DE, BC con EF y CD con FA son paralelas, y sea $p$ el semiperímtro. Sean $R_A$, $R_B$ y $R_C$ los circunradios de los triángulos $FAB$, $BCD$ y $DEF$ respectivamente. Demuestra que $R_A+R_B+R_C\geqslant p$

2- ¿Cuántos subconjuntos de ${1, 2, 3, ..., 2015}$ cumplen que la suma de sus elementos es múltiplo de $5$?

3 comentarios:

Juan dijo...
Este comentario ha sido eliminado por el autor.
Juan dijo...

Parto a mi conjunto en $403$ bloques del estilo $ 1,2,3,4,5 $. Ahora tomo un subconjunto al azar de $ 1,2,...,2015 $ y considero la probabilidad de que su suma sea múltiplo de 5.

Notemos que cada bloque tiene probabilidad de $1/4$ de sumar a un múltiplo de 5 y probabilidad $3/4$ de no hacerlo. Entonces supongamos que $A$ de mis bloques suman a un no-múltiplo de 5 y $403-A$ de mis bloques suman a un múltiplo de $5$. Sea $P_A$ la probabilidad de que si tengo $A$ bloques que NO sumen a un múltiplo de 5 cada uno, entonces la suma de TODOS ELLOS sí sea un múltiplo de 5. Entonces la probabilidad es

$\displaystyle\sum_{A=0}^{403} \displaystyle\frac{{{403}\choose{A}}3^AP_A}{4^{403}}$.

Pero puedo ver a $P_A$ como la probabilidad de lo siguiente: si tengo $A$ números al azar electos del conjunto $\{1,2,3,4\}$; cada uno con la misma probabilidad, entonces la suma de todos es multiplo de 5. Claramente $P_0=1,P_1=0$ y $P_{A+1}=\frac{1-P_A}{4}$, y usando esa fórmula recursiva $A$ veces, obtengo que

$P_A = \displaystyle\frac{1}{5} + \displaystyle\frac{(-1)^A}{5*4^{A-1}}$

Y entonces mi probabilidad se convierte en

$\displaystyle\sum_{A=0}^{403} \displaystyle\frac{{{403}\choose{A}}3^A}{5*4^{403}}+$


$\displaystyle\sum_{A=0}^{403} \displaystyle\frac{{{403}\choose{A}}3^A(-1)^A}{4^{403}*5*4^{A-1}}$,

lo cual es

$\displaystyle\sum_{A=0}^{403} \displaystyle\frac{ (3+1)^{403} + 4 \cdot \left( 1-\frac{3}{4} \right)^403 }{5*4^{403}}$,

lo cual es

$\displaystyle\frac{1}{5} + \displaystyle\frac{4}{5*4^{2*403}}$.

Multiplicando por $2^{2015}$ (el número de subconjuntos), obtengo que la respuesta es:

$\displaystyle\frac{2^{2015}+2^{405}}{5}$.

Unknown dijo...

Sugerencia para el 5 de 1996.
Considerar los rectángulos que cubren al hexágono y que tocan todos sus vértices.

Publicar un comentario